LSAT and Law School Admissions Forum

Get expert LSAT preparation and law school admissions advice from PowerScore Test Preparation.

User avatar
 Dave Killoran
PowerScore Staff
  • PowerScore Staff
  • Posts: 5853
  • Joined: Mar 25, 2011
|
#23498
Complete Question Explanation

Assumption-SN. The correct answer choice is (D)

This stimulus contains several pieces, but one key piece is missing. This sets up a Supporter Assumption perfectly, but the problem is not easy due to the abstract terms used. The argument breaks down as follows:

  • Premise (1st sentence): ..... Enact laws that benefit :arrow: Consider consequences

    Premise (3rd sentence): ..... Repugnance/Enthusiasm

    Conclusion (2nd sentence): ..... Enact laws that benefit
Note that the conclusion is the negation of the sufficient condition of the first sentence. This is actually beneficial from an information standpoint since it means we need to enact a contrapositive at some point to produce this negated condition. Thus, to solve the problem, we'll need to link terms and then have a negation of the "consider consequences" condition along the way (which will enact the contrapositive). From this angle, the problem works as follows:

  • Simple version:

    Premise: A :arrow: B
    Assumption: B

    Conclusion: A




    True version:

    Premise: A :arrow: B
    Premise: C
    Assumption: C :arrow: B

    Conclusion: A
Consequently, to link the terms properly, the answer we are seeking is Repugnance/Enthusiasm :arrow: Consider consequences, or its contrapositive. This mirrors the C :arrow: B assumption shown in the second example above. With that in mind, you can search for the answer which matches that relationship, which in this case is (D). Each of the other answers does not "fill the gap" in this Supporter Assumption, and is thus incorrect.

Answer choice (D): This is the correct answer choice. As noted above, we are seeking the following answer: Repugnance/Enthusiasm :arrow: Consider consequences. This answer matches that perfectly, and is correct.
 moshei24
  • Posts: 465
  • Joined: Mar 20, 2012
|
#4699
I've gone over this question a bunch of times and read the explanation, and it keeps looking like a justify the conclusion question. The assumption isn't necessary given the conclusion, and if you negate the correct answer choice, it doesn't destroy the argument. I did get this question right my first time looking at it, but I don't see how it's an assumption question. I've been having some trouble at times differentiating between assumption questions and justify questions, so any help with that would be great.

Thanks,
Moshe
 Nikki Siclunov
PowerScore Staff
  • PowerScore Staff
  • Posts: 1362
  • Joined: Aug 02, 2011
|
#4705
Hi Moshe,

Thanks for your question. Sometimes Supporter Assumption questions will produce similar answers to Justify questions: the link between the premises and the conclusion will be both sufficient to justify the conclusion, and required if that conclusion were ever to hold true. This is the case here.

You are wrong to suggest, however, that the logical opposite of answer choice (D) does not weaken the conclusion. It does. If the legislators DID consider the consequences of the proposed laws, it is entirely possible that these laws could benefit constituents because considering the consequences was a necessary condition for such benefit, according to the first sentence of the stimulus. Clearly, mere consideration of these consequences is not responsive to the issue of lacking beneficial legislation.

Hope this helps!
 moshei24
  • Posts: 465
  • Joined: Mar 20, 2012
|
#4706
Right, it does weaken it, but it doesn't destroy it, which is the point of using the negation technique, correct?
User avatar
 Dave Killoran
PowerScore Staff
  • PowerScore Staff
  • Posts: 5853
  • Joined: Mar 25, 2011
|
#4707
Hey Moshe,

A few points I'd like to add here (that said, I haven't referenced this question specifically; I just want to address the question you asked in the last post):

"Destroy" is a relative term, and what you might perceive as just a slight weakening might be seen as a dismantling by someone else.

Typically, the assumptions they choose as answers will have a very destructive effect when negated, but they don't have to. I could choose a very peripheral assumption that would feel "weaker" than other assumptions when negated. For LSAT purposes, if the argument is attacked when the answer is negated, that will be the correct answer. If you see multiple answers you think weaken the argument after negation, in my experience the student is wrongly interpreting one of the answers.

Thanks!
 moshei24
  • Posts: 465
  • Joined: Mar 20, 2012
|
#4708
Like I said before, I chose the correct answer the first time, because it was the only one that worked. In the past, I was told on the forum that an assumption question would destroy the stimulus when negated. Unless, I was misunderstanding. But the point I'm trying to make is that in this particular question, the answer CANNOT use the negation technique because it doesn't cause an accurate causal reasoning. It would be a mistaken negation, I believe, to negate the answer choice and come a negated conclusion. That's how that technique works - negating the answer choice destroys the conclusion. If it only weakens the conclusion, you CANNOT use that technique, which is what I was saying about this problem.

If the way these questions work is that an assumption question can have an answer that would work for the exact same question being made into a justify question, then it all makes sense. If the stem is what decides if it's an assumption question or a justify question then it works. Do you see what I'm getting at?
 moshei24
  • Posts: 465
  • Joined: Mar 20, 2012
|
#4709
In this particular question, the assumption make the conclusion necessary. Isn't that the exact form of a justify question?

Isn't the conclusion supposed to make the assumption necessary in an assumption question?
 moshei24
  • Posts: 465
  • Joined: Mar 20, 2012
|
#4710
And negating answer choice D does not hurt the argument. "considering the consequences does not result in them enacting beneficial laws." The opposite of that is true.

And just b/c arousing repugnance or enthusiasm results in them not enacting beneficial laws does not mean that it's necessary that arousing repugnance or enthusiasm means the lawmakers don't consider consequences. That would justify the conclusion - if that assumption was true then the conclusion would be true, but not the other way around.

What am I missing here?
 Nikki Siclunov
PowerScore Staff
  • PowerScore Staff
  • Posts: 1362
  • Joined: Aug 02, 2011
|
#4711
A word of caution about arguing with the LSAT: you will always lose. Use the methods you learn, but be aware of context. Your decision as to which answer to choose should ultimately be based on a comparative analysis of all five answer choices, and blindly applying techniques and rules without awareness of context will lead you down the wrong path.
 moshei24
  • Posts: 465
  • Joined: Mar 20, 2012
|
#4712
Right, I see. That's why I chose that answer.

But you're saying that the LSAT will not always go by the book and at times I'll need to improvise based on the context, right? Sometimes it will be a little bit more complicated, correct?

Get the most out of your LSAT Prep Plus subscription.

Analyze and track your performance with our Testing and Analytics Package.